Xavier is building a triangular fence the length of the second side will be 1 foot longer than the length of the first side the length of the third side will be 1 foot longer than the length of the second side the total perimeter of the triangle will be 63 feet what will the length of the third side B

Answers

Answer 1

Answer:

22

Step-by-step explanation:

Xavier Is Building A Triangular Fence The Length Of The Second Side Will Be 1 Foot Longer Than The Length

Related Questions

is angle 2 in the diagram and angle of elevation or an angle of depression​

Answers

Answer:

Angle of elevation

Step-by-step explanation:

An angle measured from the horizontal upwards is an angle of elevation.

An angle measured from the horizontal downwards is an angle of depression.

∠ 2 is a measure from the horizontal upwards thus angle of elevation.

Note that ∠ 3 is an angle of depression


How many points do you need in order to name a ray?
O
one
four
о
three
two

Answers

Answer: D) Two

You need the starting point, and another point that helps direct where the ray is aimed. Call these two points A and B.

Saying "Ray AB" means we extend a line through AB such that the line goes forever through B and beyond B, but we do not do the same for point A. The point A is effectively a cliff where no road goes on the other side of it. Check out the diagram below to see what I mean. The arrow means the line goes on forever in that direction.

What the answer to (2/3) ÷ (3/4)

Answers

Answer:

0.8888

Step-by-step explanation:

Step-by-step explanation:

Remember: when we divide fractions, we always:

• Flip the sign (change it to a multiplication sign)

• Flip the second fraction (4 goes on top and 3 goes on the bottom)

Now we have the rules covered, let's set up our expression.

[tex]\frac{2}{3} *\frac{4}{3}[/tex]

To multiply, we just have to multiply across. 2 • 4 is 8, and 3 • 3 is 9.

Our final answer: [tex]\frac{8}{9}[/tex]

I hope this helps you!

1. Morgan works as a chef at a local restaurant. She makes $15.50 per hour
She worked 30 hours this week. How much money did she earn?

Answers

Answer:

$465 for that week

Step-by-step explanation:

Since Morgan makes $15.50 an hour and she works for 30 hours the answer would be multiplication which would be this:

$15.50 × 30 = $465

Hope this Helps :)

Answer:

she earned $465

Step-by-step explanation:

you multly the money by the hours

$15.50 x 30= 465

Please helppppppppppppp

Answers

Answer:

the 5th diagram illustrates

The parallelogram shown below has an area of 54 units^2 squared. Find the missing height. h = units.

Answers

Answer:

See Explanation

Step-by-step explanation:

This question requires an attachment.

However, if you follow my explanation, you'll arrive at your answer

The area of a parallelogram is:

[tex]Area (A)= Base (b) * Height (h)[/tex]

[tex]A = b * h[/tex]

To solve for Height, we simply make h the subject of formula

[tex]h = \frac{A}{b}[/tex]

In this question, we have that A = 54

Now, let's assume base (b) = 6.

This implies that

[tex]h = \frac{54}{6}[/tex]

[tex]h = 9[/tex]

Or;

Assume that b = 3

[tex]h = \frac{54}{3}[/tex]

[tex]h = 18[/tex]

So, whatever the base of the parallelogram is;

You can easily solve for the height using the above steps

Please help, enter your answer as a number to a power!

Answers

●✴︎✴︎✴︎✴︎✴︎✴︎✴︎✴︎❀✴︎✴︎✴︎✴︎✴︎✴︎✴︎✴︎✴︎●

Hi my lil bunny!

❧⎯⎯⎯⎯⎯⎯⎯⎯⎯⎯⎯⎯⎯⎯⎯⎯⎯⎯⎯⎯⎯⎯⎯⎯⎯⎯⎯⎯⎯⎯⎯⎯⎯⎯⎯⎯⎯⎯☙

Let's simplify step by step.

[tex]9x^{-53}[/tex] × [tex]9x^{37}[/tex]

[tex]= ( 9x^{-53}[/tex] × [tex]9x^{37})[/tex]

[tex]= 9x^{-53}[/tex] × [tex]9x^{37}[/tex]

[tex]= 9x^{-16}[/tex]

[tex]= \frac{1}{_{9} 16}[/tex]

[tex]= \frac{1}{1853020188851841}[/tex] (Decimal: 0)

❧⎯⎯⎯⎯⎯⎯⎯⎯⎯⎯⎯⎯⎯⎯⎯⎯⎯⎯⎯⎯⎯⎯⎯⎯⎯⎯⎯⎯⎯⎯⎯⎯⎯⎯⎯⎯⎯⎯☙

●✴︎✴︎✴︎✴︎✴︎✴︎✴︎✴︎❀✴︎✴︎✴︎✴︎✴︎✴︎✴︎✴︎✴︎●

Have a great day/night!

❀*May*❀

help me pleasee (and no I’m not gonna follow u if u ask just help)

Answers

Answer:

option B is correct -7 is the answer

Step-by-step explanation:

=4⁴-3³.(12-3)-20

=256-27×(9)-20

=256-243-20

=256-263

=-7

hope it will help ^_^

Answer:

[See Below]

Step-by-step explanation:

First you'll solve the parenthesis part:

* 12 - 3 = 9

Then Solve the exponents:

* 4^4 = 256 (4 x 4 x 4 x 4)

* 3^3 = 27 (3 x 3 x 3)

Rewrite:

* 256 - 27 x 9 - 20

Multiply:

* 27 x 9 = 243

Subtract:

* 256 - 243 = 13

* 13 - 20 = -7

So the answer would be B, -7

(Remember to use PEMDAS to solve)

Find the value of term a14 in the sequence. 7, 2, -3, -8, -13...

Answers

Answer:

-58

Step-by-step explanation:

The first term (a₁) is 7 and the common difference (d) is 2 - 7 = -5.

Explicit formula of arithmetic sequence: aₙ = a₁ + (n - 1) * d

aₙ = 7 + (n - 1) * (-5) = 7 - 5n + 5 = -5n + 12

Therefore, a₁₄ = -5 * 14 + 12 = -58.

What is the greatest common factor of 36, 34, and 22?

Answers

factors of 36: 1,2,3,4,6,9,12,18,36
factors of 34: 1,2,17,34
factors of 22: 1,2,11,22
GCF: 2

We have that the factors of the individual Number  

(1,2,3,4,6,9,12,18,36),( 1,2,17,34),(  1,2,11,22 ) respectively

The greatest common factor is 2

2

From the question we are told

What is the greatest common factor of 36, 34, and 22?

Generally the equation for the Factors  is mathematically given as

36 factors are as follows 1,2,3,4,6,9,12,18,36

34 factors  factors are as follows  1,2,17,34

22 factors factors are as follows   1,2,11,22

Therefore

From the factors of the individual Number  

(1,2,3,4,6,9,12,18,36),( 1,2,17,34),(  1,2,11,22 ) respectively

The greatest common factor is 2

2

For more information on this visit

https://brainly.com/question/14209188?referrer=searchResults

For the following questions, solve for the missing rate, base or product (round to the nearest hundredth if needed): 25% of 320 is

Answers

Answer:

80

Step-by-step explanation:

let the unknown value be x.

[tex]\frac{25}{100}*320=x\\ \frac{25*320}{100}=x\\ \frac{8000}{100}=x\\ 80=x[/tex]

Answer:

[tex]\huge\boxed{80}[/tex]

Step-by-step explanation:

[tex]25 \% \ of \ 320[/tex]

% means out of hundred and of means to multiply.

=> [tex]\\\frac{25}{100} * 320\\[/tex]

=> 0.25 * 320

=> 80

Which of the following is a correct comparison of two rational numbers?a. 1.5 > 5/3b. 2.5 < 0.25c. 0.07 > 7/10d. 0.6 > 4/7

Answers

Answer:

d. 0.6 > 4/7

Step-by-step explanation:

a. 1.5 > 5/3

From part a, it is given that 1.5 is greater than 5/3

changing 5/3 to decimal figure, we get 1.67

Thus, the claim that 1.5 > 1.67 is not valid because 1.67 is greater than 1.5

b. 2.5 < 0.25

given that 2.5 is less than 0.25 is not valid because 2.5 is greater than 0.25

c.  0.07 > 7/10

if we convert 7/10 to decimal figure, we get 0.7

the claim that 0.07 is greater than 0.7 is not valid because 0.7 is greater than 0.07

d. 0.6 > 4/7

If we convert 4/7 to decimal figure, we get  0.57

Therefore, the claim that 0.6 is greater than 0.57 is accurate and valid

Hence, Option d is the correct answer.

a manned rocket accelerates at a rate of 20 m/s2 during launch How long does it take the rocket reach a velocity of 400 m/s?

Answers

Answer:

Step-by-step explanation:

acceleration, a = 20 m/s^2

initial velocity, u = 0 since it starts from rest

final velocity, v = 400 m/s

v = u + at

400 = 0 + 20 t

400 = 20 t

t = 400 / 20

t = 20 sec

Hope this helps

plz mark as brainliest!!!!

It will take 20 sec the rocket reach a velocity of 400 m/s

What are equations of motion?

There are three equation of motions that can be used when motion of the object is under constant acceleration and on a straight path.

They are listed below as:

[tex]v = u + at\\\\s = ut + \dfrac{1}{2} at^2\\\\v^2 = u^2 + 2as[/tex]

We are given that manned rocket accelerates at a rate of 20 m/s2 during launch

The acceleration, a = 20 m/s^2

The initial velocity, u = 0 since it starts from rest

The final velocity, v = 400 m/s

Therefore, v = u + at

400 = 0 + 20 t

400 = 20 t

t = 400 / 20

t = 20 sec

Hence, It will take 20 sec the rocket reach a velocity of 400 m/s

Learn more about equation of motion here:

https://brainly.com/question/5955789

#SPJ2

Find the equation of a line that is perpendicular to the line -2y+20=8x and passes through the point (7,2).

Answers

Answer:

  8y -2 = 2x

Step-by-step explanation:

The equation of the perpendicular line will have the coefficients of x and y swapped, and one of them negated. The constant can be found by substituting the given point value:

  8y +constant = 2x . . . . coefficients swapped; former y-coefficient negated

  8(2) +constant = 2(7) . . . . . fill in given point values (x, y) = (7, 2)

  constant = 14 -16 = -2 . . . . subtract 16

The equation can be written as ...

  8y -2 = 2x

_____

Additional comment

Personally, I prefer standard form, which has common factors removed from the coefficients, and has the constant separated from the variable terms:

  x -4y = -1

Some like the slope-intercept form better:

  y = 1/4x +1/4

Apparently, your curriculum author doesn't care how the equation of the line is written, so we have answered with the same form that was given.

If k3 – 9 = 12, what is the value of k? A. 1 B. 7 C. 9 D. 63

Answers

Answer:

63

Step-by-step explanation:

The answer is 63 because k/3 =63

k/3-9=12

We move all terms to the left:

k/3-9-(12)=0

We add all the numbers together, and all the variables

k/3-21=0

We multiply all the terms by the denominator

k-21*3=0

We add all the numbers together, and all the variables

k-63=0

We move all terms containing k to the left, all other terms to the right

k=63

Brainliest please i just need 10 to move to next level

An equation is formed when two equal expressions. The value of k in the given equation is 63. The correct option is D.

What is an equation?

An equation is formed when two equal expressions are equated together with the help of an equal sign '='.

Given the equation k3-9=12.

Now, for the given equation the value of k can be found by solving the equation,

k/3 – 9 = 12

k/3 – 9 + 9 = 12 + 9

k/3 = 21

k/3 × 3 = 21 × 3

k = 63

Hence, the value of k in the given equation is 63.

Learn more about Equation here:

https://brainly.com/question/2263981

#SPJ2

Please help me on this

Answers

Answer:

-q

Step-by-step explanation:

We know that we are trying to find an expression equivelant to p - q:

p - q can be p - 1q

We can then put parenthesis around -1q:

p + (-1q)

We then can simplify:

p + (-q)

what is (5x-2)(4x^2 -3x-2)

Answers

Answer: 20x^3 - 23x^2 - 4x + 4

Explanation:

Use distributive property:

(5x-2)(4x^2 - 3x-2)
= 20x^3 - 15x^2 - 10x - 8x^2 + 6x + 4
= 20x^3 - 23x^2 - 4x + 4

Answer:

20x^3-23X^2-4X+4

Step-by-step explanation:

AB has endpoints at (-4,5) and (4,-3) using the midpoint formula what is the midpoint segment

Answers

Answer:

(0,1)

Step-by-step explanation:

What is the distance across the river in feet?

Answers

Answer:

375 ft

Step-by-step explanation:

Since the triangles are similar then the ratios of corresponding sides are equal, that is

[tex]\frac{d}{125}[/tex] = [tex]\frac{30}{10}[/tex] ( cross- multiply )

10d = 3750 ( divide both sides by 10 )

d = 375 ft

4b + 11 + 8b + 1 = 180

Answers

Answer:

b=14

Step-by-step explanation:

Add 4b and 8b, which you get 12b. Then you add 11 and 1 which you get 12.

12b + 12 = 180 is the next equation.

Afterwards, you subtract 12 from 180.

12b = 168

Divide 12b from 168.

b = 14

Find for B?

Mk in this equation

4b + 11 + 8b + 1 = 180

Subtract 11 and 1 from both sides

4b + 8b = 168

Combine unknowns

12b = 168

Find b by dividing by 12

Thus, B = 14

Hope this helps,

Jeron

P.S, if you found this really helpful, consider marking brainliest, (you don't have to btw)

Find the Product. 437 × 16​

Answers

Answer: [tex]6692[/tex]

Look at the image for work...

Please Help! I'm Stuck!

Answers

Answer:

19

Step-by-step explanation:

Assuming that [tex]l[/tex] represents length and [tex]w[/tex] represents width, we can make a systems of equations and solve for l and w.

[tex]2l+2w=60[/tex], since the perimeter of a rectangle will be double the length plus double the width.

Also we can make the equation [tex]l = 2w+3[/tex], as stated in the last part of the question.

We can now substitute the value of [tex]2w+3[/tex] into the equation [tex]2l+2w=60[/tex] as l.

[tex]2(2w-3) + 2w = 60\\\\4w-6+2w=60\\\\6w-6=60\\\\6w=66\\\\w = 66\div6\\\\w = 11[/tex]

So we know the width is 11. Now that we know the width, we can substitute it back into the equation  [tex]l = 2w+3[/tex] to find the length.

[tex]l=2\cdot11-3\\\\l=22-3\\\\l=19[/tex]

So the length is 19.

Hope this helped!

Danica made $440 working at a pet shop last month. She worked a total of 40 hours. How much money did Danica make per hour?​

Answers

Answer:

Danica made 11 dollars per hour.

Step-by-step explanation:

Divide- 440/40=11

Check 11*40=440

She made 11 dollars per hour.

Answer:11

Step-by-step explanation:

3(3a - 5b) - (a - 76)

Answers

Answer:

8a-15b+76

Step-by-step explanation:

that is the answer

Find ????⋅ ???? if |????|=3 , |????|=9 , and the angle between ???? and ???? is π2 radians.
a.b = _______

Answers

Answer:

[tex]a{\cdot}b=0[/tex]

Step-by-step explanation:

It is given that,

Magnitude of a A, [tex]|A|=3[/tex]

magnitude of B, [tex]|B|=9[/tex]

The angle betwern a and b is [tex]\dfrac{\pi}{2}\ rad=90^{\circ}[/tex]

Dot product,

[tex]a{\cdot} b=|a||b|\ \cos\theta\\\\a{\cdot} b=3\times 9\times \ \cos(90)\\\\a{\cdot} b=0[/tex]

So, a.b is equal to 0.

You need to multiply n by a power of 10 to help you find the fraction. Decide on the power of 10 to multiply by, and tell how you identified that number

Answers

Answer: -10

I believe it is [tex]n^{-10}[/tex] because that would make a fraction: [tex]\frac{1}{n^{10} }[/tex]

Sorry, I haven't done this stuff in a while, but I hope this helped :)

A = 5 B = 3 C = 1

6A - 10B + 3C = ?

Answers

Answer:

3

Step-by-step explanation:

6(5)-10(3)+3(1)

6*5=30

10*3=30

3*1=3

30-30+3=3

Solve 4(p - 15)=312 please

Answers

Answer:

p = 93

Step-by-step explanation:

4(p - 15)=312

4p - 60 = 312

4p - 60 + 60 = 312 + 60

4p = 372

4p/4 = 372/4

p = 93

Answer:

the answer is 93......

A vertical pole is placed in the ground at a campsite outside Salt Lake City, Utah. One winter day, 16 of the pole is in the ground, 23 of the pole is covered in snow, and 1.8 ft is above the snow. How long is the pole, and how deep is the snow

Answers

Answer:

Length of the pole = 10.8 ft

Depth of the snow = 7.2 ft

Step-by-step explanation:

The correct question is

A vertical pole is placed in the ground at a campsite outside Salt Lake City, Utah. One winter day, 1/6 of the pole is in the ground, 2/3 of the pole is covered in snow, and 1.8 ft is above the snow. How long is the pole, and how deep is the snow.

The fraction of the pole in the ground = 1/6

The fraction of the pole in the snow = 2/3

The length of the pole above the snow = 1.8 ft

Total fraction of the length of the pole = 1

For the fraction of the length above the snow, we solve as

1/6 + 2/3 + [tex]x[/tex] = 1

[tex]x[/tex] = 1 - 1/6 - 2/3 = 1/6

This means that the length of the pole above the snow 1.8 ft = 1/6 of the length of the pole.

Also, the fraction of the pole in the ground = 1/6

this means that the length of the pole in the ground = 1.8 ft

If 1/6 of the length of pole is 1.8 ft,

the length of the pole = 1.8 ft x 6 = 10.8 ft

Length of the pole in the snow = 2/3 of 10.8 = 2/3 x 10.8 = 7.2 ft

this means that the the snow is 7.2 ft deep

{(-4, 1), (-3,5), (-1,0), (6, 2), (9,5)} is this a function or not?​

Answers

Yes, it’s a function because there is no repeating x-values,
Other Questions
A(n) ____ focuses on the need for service providers to build strong bonds with customers. Which natural rights did Rousseau believe human beings are born with? life, liberty, and happiness life, liberty, and property liberty, social justice, and education liberty, social justice, and property Although Joshua is an alcoholic he is able to control his drinking when he parties with his friends T Or F 8. The very last step Saraa should take, according to the decision-making process, would be to __________. select alternatives evaluate the decision's effectiveness analyze alternative solutions implement the alternative Please help me to prove this.. Does anyone know how to solve this? Its really starting to stress me out.PS: You just can solve the lim(x->2) instead of lim(u->0) if u want A price decreases from $986 to $828.24. What is the percent change? VocabularioComplete the sentences using Vocabulary words.1. Una masa de agua que est rodeada (surrounded, de terra es un2. Laocurre cuando los bosques se destruyen.3. Una regin donde no llueve nunca o casi nunca es uri4. Las nubes y el sol estn en el5. Por el da vemos el sol; por la noche vemos la luna y las6. Laes un animal que nos da leche.7. El salmn es un tipo de8. El turismo que protege y conserva la naturaleza es el9. EIes un animal que vive en los rboles de las selvas tropicales.10. Un volcn es una montaa que tiene un11. El lugar donde vivimos es nuestro medio12. Para conservar los recursos, debemosel aluminio, el vidrio y el plstico. Terrell and hi parents live in a home near San Francisco. Terrell has a Visa card, which he is using to pay for car insurance today. Write a short paragraph telling ways Terrell and his family are making use of the U.S. monetary systems and policies. (Please help fast!) .......... In the Access Query window, the upper portion of the window contains the design grid.a) trueb) false three examples of monosaccharides Whats the difference between a run-on and a comma spice ? What was the significance of Vasco Nez de Balboas sighting of the Pacific Ocean? as Arav write a diary entry on paying a visit to old age home Assume Intervale Railway is considering investing in Pale Co. stock for three months. The investment will represent 5% of the voting stock of Pale Co. How would the investment be classified?a. Significant interest investmentb. Trading investmentc. Held-to-maturity investmentd. Controlling interest investment What is standard form for 400,000+60,000+5,000+100 What is the area of a square with the vertices (3, 3), (6, 6), (9,3), and (6,0)? There was a long line of people and all of the tellers were busy helping customers, many of whom were there to bank their paychecks. Use context clues to find the correct definition of the multiple-meaning word bank as it is used in this sentence. to deposit into an account a financial institution the side of the river to count on x+2/x2 x2/x+2 = 5/6